June 2007 - Sec 2 - LR - Q8

Video Transcript:

0:06
Question 8: Proponents of the electric car maintain that when the technical
0:11
problems associated with its battery design are solved, such cars will be widely
0:16
used and, because they are emission-free, will result in the abatement of the environmental
0:20
degradation caused by auto emissions. But unless we dam more rivers the electricity to
0:26
charge these batteries will come from nuclear or coal-fired power plants. Each of these
0:33
three power sources produces considerable environmental damage. Thus,
0:38
the electric car blank. So we notice another argument completion question. Which one of
0:45
the following most logically completes the argument. So again keep in mind what
0:53
we already know that these proponents of this electric car maintained that once
0:57
these technical problems these cars will be widely used they are emission free and
1:02
therefore will result in an abatement of the environmental degradation caused by
1:07
auto emissions. However the electricity used to charge these batteries of these
1:14
electric cars will come from sources that produced considerable environmental
1:18
damage. Thus, the electric car... So let's try to find an answer that logically completes
1:26
this argument. So again these are a type of must be true question. Keeping in mind
1:30
the information that was given to us before the blank let's take a look at (A): Will
1:35
have worse environmental consequences that its proponents may believe. Does
1:40
that logically complete the argument? And you notice that it does because the proponents are
1:45
saying that when these are widely used and because their emission free this will
1:50
result in an abatement of the environmental degradation caused by auto emissions. The
1:55
problem is however that the electricity used to charge these batteries produces
2:00
considerable environmental damage itself. Thus, the electric car will have worse
2:06
environmental consequences than its proponents may believe. And (A) would be the correct
2:11
answer. But again let's just make sure. (B) Will probably remain less popular
2:18
than other types of
2:19
cars. And you notice that we're not taking issue with the popularity of
2:24
electric cars. The fact that they're going to be widely used if the battery design
2:29
problems are solved. Were not taking issue with that. We're taking issue with the
2:33
fact that the electricity to charge these batteries causes considerable
2:36
environmental damage. So (B) does not follow. (C) Requires that purely technical
2:43
problems to be solved before it can succeed. And again that's not the issue
2:46
that's not what this passage is about. Its not about whether we're solving these
2:50
technical problems. It's even if they are solved is it going to result in an abatement of the
2:55
environmental degradation caused by auto emissions. And since the electricity
3:00
used to charge these batteries causes considerable environmental damage itself
3:04
that is what we're talking about.
3:07
Not this idea of purely technical problems being solved. So (C) is out. Bringing me to (D): Will
3:13
increase the total level of emissions rather than reduce it. And you notice
3:18
that is too harsh
3:19
we don't know if it's gonna increase it. It's possible that is just not as big of
3:25
a decrease that its proponents thought. So (D) is too harsh. Again remember these are must be true
3:31
questions. If it follows logically it must be true based on the information before. It
3:37
doesn't tell us whether there is any environmental damage caused by the
3:40
electricity to charge these batteries is more than the reduction in auto emissions.
3:46
It doesn't say anything about that. So these are too harsh. And bringing me lastly to (E):
3:51
Will not produce a net reduction in environmental degradation. Again we don't
3:58
know. All we know is that electric cars are mission free and if they're widely used
4:05
clearly we're going to have a drop in environmental damage. However, the
4:11
electricity used to charge these batteries causes considerable damage. But again we don't
4:16
know the relative size of either of these groups. So (E) does not follow
4:20
logically and (E) would be eliminated. (D) and (E) have the same problem. It will increase will
4:25
not produce a net reduction. We don't have information about that in the
4:28
passage. Whereas (A) much more general,
4:32
much more easy to support 100% and again 'logically completes' implies that it must be true
4:38
so (A) would be the correct answer.